Đến nội dung

Hình ảnh

$ \boxed{TOPIC} $ Các bài toán hình học hướng đến Olympic

* * * * * 3 Bình chọn

  • Please log in to reply
Chủ đề này có 9 trả lời

#1
Sin99

Sin99

    Thượng sĩ

  • Thành viên
  • 238 Bài viết

$ \boxed{ \text{Giới thiệu}} $ Chào các bạn, khát khao của mình sau khi đậu chuyên là sẽ tích cực hoạt động ở VMF, và cho đến bây giờ, mục tiêu thứ nhất xem như đã hoàn thành, đến với mục tiêu thứ 2  :icon6: Để nhằm hiện thực khát khao ấy, cũng như nâng cao kiến thức, kĩ năng giải hình học phẳng, mình quyết định mở 1 Topic nhỏ với đối tượng hướng đến các bạn đam mê hình học, có nguyên vọng chinh phục các kì thi HSG ở bậc THPT từ lớp 10 đến lớp 12.

 

$ \boxed{ \text{Nội dung}} $ Như đã nói như trên, topic sẽ xoay quanh các bài toán hình học phẳng qua các mức độ dễ đến khó. Mỗi các nhân sẽ post 1 bài toán đề nghị do sưu tầm, sáng tác,... với điều kiện đưa ra lời giải của bài toán trước đó. Topic sẽ không giới hạn số lời giải của 1 bài toán. Các bạn sau khi post lời giải cần post bài toán đề nghị và cứ như thế... đến khi đơm hoa kết trái, mình sẽ tổng hợp thành file và lưu lại sau này. Do đây mới là topic nhỏ nên độ khó của các bài toán cũng ở mức giới hạn, dành cho cả HS vừa vào lớp 10.  :D Topic này được mình tham khảo từ Topic Marathon số học Olympic  :wub:

 

$ \boxed{\text{Qui định}} $

1) Các bài toán được ghi nguồn rõ ràng

2) Nội dung xoay quanh hình học phẳng, nhằm luyện tập hướng đến các kì thi Olympic.

3) Lời giải rõ ràng, cụ thể, không lan man,...

4) Không được đăng các bài toán vẫn đang trong thời gian diễn ra của những cuộc thi khác ( VD: Toán học và tuổi trẻ, toán tuổi thơ,...)

5) Bạn giải bài thứ $ n $ thì đề xuất luôn bài toán thứ  $ n +1 $

6) Tuyệt đối không spam, quảng cáo, mình sẽ báo cáo các anh quản trị viên có hình phạt thích hợp.

7) Sau 3 ngày bài toán không có lời giải, người đề cử sẽ đăng bài toán đó và tiếp tục đề cử bài mới. 

 

(P/s) Do đây là lần đầu, cộng thêm kiến thức còn rất hạn hẹp, nên có sai sót gì mong các anh chị, các bạn bỏ qua, cùng nhau đóng góp để topic và cá nhân mình ngày càng hoàn thiện.   :icon6:

 

Mình xin đề xuất bài toán đầu tiên: 

$ \boxed{\text{Bài toán 1}} $ (Gặp gỡ Toán học 2010)

Đường tròn $ (I) $ nội tiếp tam giác $ ABC $ tiếp xúc các cạnh $ AB, AC $ tương ứng tại  $ D , E $. $ P $ là 1 điểm bất kì trên cung lớn $ \mathop DE $ của $ (I) $. Lấy $ F $ là điểm đối xứng với $ A $ qua $ PD $ và $ M $ là trung điểm đoạn $ DE $. Chứng minh rằng $ \angle FMP  = 90^{\circ} $. 

Hình gửi kèm

  • Untitled.png

Bài viết đã được chỉnh sửa nội dung bởi Sin99: 23-07-2019 - 12:33


#2
Arthur Pendragon

Arthur Pendragon

    Trung sĩ

  • Thành viên
  • 134 Bài viết

AP cắt $(I)$ tại K. Q đối xứng với K qua AM. FP cắt $(I)$ tại $R$

Do $AM.AI=AK.AP=AD^2$ nên KMIP nội tiếp. Do đó $\widehat{KMP}=\widehat{KIP}$ suy ra $\widehat{KMP}+\widehat{KMA}+\widehat{AMQ}=\widehat{KIP}+2\widehat{KPI}=180^0$ nên P, M, Q thẳng hàng.

Do vậy nên ta có: $\frac{DM}{MP}=\frac{ME}{MP}=\frac{QE}{DP}=\frac{DK}{DP}=\frac{DA}{PA}=\frac{DA}{FP}$ (1)

Mặt khác,do F đối xứng với A qua DP nên DP là phân giác $\widehat{RPK}$ suy ra $DR=DK=QE$ và $DE=RQ$

Do đó: $\widehat{RPM}=\widehat{ADM}$ (2)

Từ (1) và (2) suy ra tam giác FPM đồng dạng với tam giác ADM nên $\widehat{FMP}=\widehat{AMD}=90^0$.

Bài 2: Cho tam giác $ABC$ và $(O)$ là đường tròn ngoại tiếp. Một đường thẳng d bất kỳ đi qua $O$ cắt $AB,AC$ lần lượt tại $M$ và $N$. Gọi $I,P,Q$ lần lượt là trung điểm của $MN,BN,CM$. Chứng minh 4 điểm $O,I,P,Q$ cùng nằm trên một đường tròn.

P/s: Ai biết xuất xứ bài toán này không cho mình biết với....

 


Bài viết đã được chỉnh sửa nội dung bởi Arthur Pendragon: 23-07-2019 - 08:14

"WHEN YOU HAVE ELIMINATED THE IMPOSSIBLE, WHATEVER REMAINS, HOWEVER IMPROBABLE, MUST BE THE TRUTH"

-SHERLOCK HOLMES-             


#3
Sin99

Sin99

    Thượng sĩ

  • Thành viên
  • 238 Bài viết

AP cắt $(I)$ tại K. Q đối xứng với K qua AM. FP cắt $(I)$ tại $R$

Do $AM.AI=AK.AP=AD^2$ nên KMIP nội tiếp. Do đó $\widehat{KMP}=\widehat{KIP}$ suy ra $\widehat{KMP}+\widehat{KMA}+\widehat{AMQ}=\widehat{KIP}+2\widehat{KPI}=180^0$ nên P, M, Q thẳng hàng.

Do vậy nên ta có: $\frac{DM}{MP}=\frac{ME}{MP}=\frac{QE}{DP}=\frac{DK}{DP}=\frac{DA}{PA}=\frac{DA}{FP}$ (1)

Mặt khác,do F đối xứng với A qua DP nên DP là phân giác $\widehat{RPK}$ suy ra $DR=DK=QE$ và $DE=RQ$

Do đó: $\widehat{RPM}=\widehat{ADM}$ (2)

Từ (1) và (2) suy ra tam giác FPM đồng dạng với tam giác ADM nên $\widehat{FMP}=\widehat{AMD}=90^0$.

Bài 2: Cho tam giác $ABC$ và $(O)$ là đường tròn ngoại tiếp. Một đường thẳng d bất kỳ đi qua $(O)$ cắt $AB,AC$ lần lượt tại $M$ và $N$. Gọi $I,P,Q$ lần lượt là trung điểm của $MN,BN,CM$. Chứng minh 4 điểm $O,I,P,Q$ cùng nằm trên một đường tròn.

P/s: Ai biết xuất xứ bài toán này không cho mình biết với....

 

Bạn xem lại đề bài 2, mình thấy có vấn đề vì khi vẽ có thể rơi vào trường hợp O,I,P,Q là tứ giác lõm ... 



#4
Arthur Pendragon

Arthur Pendragon

    Trung sĩ

  • Thành viên
  • 134 Bài viết

Bạn xem lại đề bài 2, mình thấy có vấn đề vì khi vẽ có thể rơi vào trường hợp O,I,P,Q là tứ giác lõm ... 

Mình thêm giả thiết tam giác ABC nhọn rồi đó.


"WHEN YOU HAVE ELIMINATED THE IMPOSSIBLE, WHATEVER REMAINS, HOWEVER IMPROBABLE, MUST BE THE TRUTH"

-SHERLOCK HOLMES-             


#5
Sin99

Sin99

    Thượng sĩ

  • Thành viên
  • 238 Bài viết

Mình thêm giả thiết tam giác ABC nhọn rồi đó.

Nhọn hay tù mình vẽ hình vẫn không ra, trừ phi bạn nói $ d $ đi qua  điểm  $ O $ 



#6
Arthur Pendragon

Arthur Pendragon

    Trung sĩ

  • Thành viên
  • 134 Bài viết

Nhọn hay tù mình vẽ hình vẫn không ra, trừ phi bạn nói $ d $ đi qua  điểm  $ O $ 

Ừ thì đúng là nó đi qua điểm $O$. Lỗi đánh máy, sorry nha  :D  :D  :D


"WHEN YOU HAVE ELIMINATED THE IMPOSSIBLE, WHATEVER REMAINS, HOWEVER IMPROBABLE, MUST BE THE TRUTH"

-SHERLOCK HOLMES-             


#7
Vitamin Love

Vitamin Love

    Lính mới

  • Thành viên mới
  • 3 Bài viết

AP cắt $(I)$ tại K. Q đối xứng với K qua AM. FP cắt $(I)$ tại $R$

Do $AM.AI=AK.AP=AD^2$ nên KMIP nội tiếp. Do đó $\widehat{KMP}=\widehat{KIP}$ suy ra $\widehat{KMP}+\widehat{KMA}+\widehat{AMQ}=\widehat{KIP}+2\widehat{KPI}=180^0$ nên P, M, Q thẳng hàng.

Do vậy nên ta có: $\frac{DM}{MP}=\frac{ME}{MP}=\frac{QE}{DP}=\frac{DK}{DP}=\frac{DA}{PA}=\frac{DA}{FP}$ (1)

Mặt khác,do F đối xứng với A qua DP nên DP là phân giác $\widehat{RPK}$ suy ra $DR=DK=QE$ và $DE=RQ$

Do đó: $\widehat{RPM}=\widehat{ADM}$ (2)

Từ (1) và (2) suy ra tam giác FPM đồng dạng với tam giác ADM nên $\widehat{FMP}=\widehat{AMD}=90^0$.

Bài 2: Cho tam giác $ABC$ và $(O)$ là đường tròn ngoại tiếp. Một đường thẳng d bất kỳ đi qua $O$ cắt $AB,AC$ lần lượt tại $M$ và $N$. Gọi $I,P,Q$ lần lượt là trung điểm của $MN,BN,CM$. Chứng minh 4 điểm $O,I,P,Q$ cùng nằm trên một đường tròn.

P/s: Ai biết xuất xứ bài toán này không cho mình biết với....

 

 

AP cắt $(I)$ tại K. Q đối xứng với K qua AM. FP cắt $(I)$ tại $R$

Do $AM.AI=AK.AP=AD^2$ nên KMIP nội tiếp. Do đó $\widehat{KMP}=\widehat{KIP}$ suy ra $\widehat{KMP}+\widehat{KMA}+\widehat{AMQ}=\widehat{KIP}+2\widehat{KPI}=180^0$ nên P, M, Q thẳng hàng.

Do vậy nên ta có: $\frac{DM}{MP}=\frac{ME}{MP}=\frac{QE}{DP}=\frac{DK}{DP}=\frac{DA}{PA}=\frac{DA}{FP}$ (1)

Mặt khác,do F đối xứng với A qua DP nên DP là phân giác $\widehat{RPK}$ suy ra $DR=DK=QE$ và $DE=RQ$

Do đó: $\widehat{RPM}=\widehat{ADM}$ (2)

Từ (1) và (2) suy ra tam giác FPM đồng dạng với tam giác ADM nên $\widehat{FMP}=\widehat{AMD}=90^0$.

Bài 2: Cho tam giác $ABC$ và $(O)$ là đường tròn ngoại tiếp. Một đường thẳng d bất kỳ đi qua $O$ cắt $AB,AC$ lần lượt tại $M$ và $N$. Gọi $I,P,Q$ lần lượt là trung điểm của $MN,BN,CM$. Chứng minh 4 điểm $O,I,P,Q$ cùng nằm trên một đường tròn.

P/s: Ai biết xuất xứ bài toán này không cho mình biết với....

 

Kẻ đường kính BE CF.
Ta có NE // OP  MF // OQ
=> ∠EKF = ∠POQ (1)
Lại có K thuộc (O) theo định lí pascal ( K là giao của EN và FM )
=> ∠EKF = ∠BAC = ∠PIQ (2)
Từ (1) và (2) => ∠PIQ = ∠EKF => ĐPMC
( bài này xuất phát từ IMO 2009 )
Bài toán 3: Cho tam giác ABC nhọn nt (O) đường tròn (K) đi qua BC cắt AC AB tại E F, BE CF cắt tại H. P là điểm tùy ý trên AH. (APB) (APC) cắt BE CF tại M, N. X, Y, Z theo thứ tự là tâm đường tròn (APB) (APC) (BMC). Q, R là tâm (XYZ) (MNP). Chứng minh: QR // AP, (XYZ) (MNP) đồng tâm (khi K thuộc BC)


#8
Sin99

Sin99

    Thượng sĩ

  • Thành viên
  • 238 Bài viết

Bài toán 2 còn có 1 cách giải sử dụng đường trung bình. 

Kẻ đường kính $ AK $ của $ (O) $. Gọi $ H $ là trực tâm $ \Delta AMN $, $ T $ là trung điểm $ HK $.

Dễ thấy $  NH // BK $ ( cùng $ \bot AB $ ) mà $ P, T $ là trung điểm $ BN, HK $ nên $ NH // PT // BK $ 

Mặt khác $ IP // AB $ suy ra $ PT \bot IP $. 

Chứng minh tương tự ta có $  OT \bot MN $ , $ IQ \bot QT $ 

Vậy 5 điểm $ I, O , Q , T, P $ đồng viên. 

Suy ra điều phải chứng minh. 

Mở rộng : $ \Delta ABC $, $ d $ bất kì đi qua $ ( O ) $ cắt   $ ( O ) , AB, AC $ lần lượt tại $ X, Y, M, N $. Gọi $ J, I , P , Q $ là trung điểm $ XY, MN, BN, CM $. Khi đó $ JIQP $ nội tiếp. 

 

 

( P/S mình bổ sung thêm qui định đó là sau 3 ngày bài toán đề xuất không có lời giải thì người up có thể up lời giải cho mn cùng tham khảo, tránh tình trạng bí cục bộ  :lol:, người up vẫn sẽ đề xuất bài tiếp theo. )


Bài viết đã được chỉnh sửa nội dung bởi Sin99: 30-07-2019 - 13:38


#9
Vitamin Love

Vitamin Love

    Lính mới

  • Thành viên mới
  • 3 Bài viết

Bài toán 2 còn có 1 cách giải sử dụng đường trung bình. 

Kẻ đường kính $ AK $ của $ (O) $. Gọi $ H $ là trực tâm $ \Delta AMN $, $ T $ là trung điểm $ HK $.

Dễ thấy $  NH // BK $ ( cùng $ \bot AB $ ) mà $ P, T $ là trung điểm $ BN, HK $ nên $ NH // PT // BK $ 

Mặt khác $ IP // AB $ suy ra $ PT \bot IP $. 

Chứng minh tương tự ta có $  OT \bot MN $ , $ IQ \bot QT $ 

Vậy 5 điểm $ I, O , Q , T, P $ đồng viên. 

Suy ra điều phải chứng minh. 

Mở rộng : $ \Delta ABC $, $ d $ bất kì đi qua $ ( O ) $ cắt   $ ( O ) , AB, AC $ lần lượt tại $ X, Y, M, N $. Gọi $ J, I , P , Q $ là trung điểm $ XY, MN, BN, CM $. Khi đó $ JINM $ nội tiếp. 

 

 

( P/S mình bổ sung thêm qui định đó là sau 3 ngày bài toán đề xuất không có lời giải thì người up có thể up lời giải cho mn cùng tham khảo, tránh tình trạng bí cục bộ  :lol:, người up vẫn sẽ đề xuất bài tiếp theo. )

Trung điểm XY ?? PQ có bị thừa ??



#10
Sin99

Sin99

    Thượng sĩ

  • Thành viên
  • 238 Bài viết

Trung điểm XY ?? PQ có bị thừa 

À t ghi nhầm $ JIQP $ không phải $ JIMN $    :ukliam2:


Bài viết đã được chỉnh sửa nội dung bởi Sin99: 30-07-2019 - 13:38





0 người đang xem chủ đề

0 thành viên, 0 khách, 0 thành viên ẩn danh